You are on page 1of 32

Q.

1) Ans: a Exp:
Exp: • Statement 1 is correct: The Inter-State Council
● Statement 1 is correct : Trade restrictions can be was established under Article 263 of the
imposed by parliament to safeguard public Constitution of India through a Presidential Order
interest howeve, no discrimination can be made dated 28th May 1990. The Council consists of -
between states while imposing restrictions . ▪ Prime Minister – Chairman
o The free flow of trade, commerce, and ▪ Chief Ministers of all States – Members
intercourse within and across inter-State ▪ Chief Ministers of Union Territories having a
borders is an important prerequisite for Legislative Assembly and Administrators of UTs
ensuring economic unity, stability, and not having a Legislative Assembly – Members
prosperity in a two-tier polity country. ▪ Six Ministers of Cabinet rank in the Union
o The Indian Constitution includes provisions that Council of Ministers to be nominated by the
guarantee freedom of inter-state trade and Prime Minister – Members
commerce throughout India's territory. • A Standing Committee of the Inter-State Council
● Statement 2 is incorrect : The State legislature has been constituted for continuous consultation
can impose reasonable restrictions on the and processing of matters for the consideration of
freedom afforded by Art 301 to safeguard public the Council.
interests. A bill for this purpose can be introduced o The Standing Committee comprises Union Home
in the legislature only after prior Minister as Chairman and 5 Union Ministers of
recommendation of the president . Cabinet Rank and 9 Chief Ministers of States as
Members nominated by the Chairman of the
Q.2) Ans: b Inter-State Council.
Exp: • Statement 2 is Incorrect : Five union cabinet
● Article 1 of the Indian Constitution declares that ministers and nine chief ministers are part of this
India, that is, Bharat is a Union of states and not committee. Main function of this committee to
federation of states. This article deals with two promote center state relations and to ensure
things - Name of the country and type of polity. better coordination among states.
● There were various opinions in the Constituent
Assembly with regard to the name of the country. Q.4) Ans: d
Some members suggested the traditional name Exp:
(Bharat), while others advocated the modern ● Statement 1 is incorrect: The Inter state council
name (India). Hence, the Constituent Assembly was established by presidential order on the
had to adopt a mix of both (‘India, that is, recommendation of the Sarkaria committee in
Bharat’) 1990.
● The country is described as ‘Union’ although its o Inter-State Council is a recommendatory body
Constitution is federal in structure. with duties to investigate and discuss the
● According to Dr. B.R. Ambedkar, the phrase subjects of common interest between the Union
‘Union of States’ has been preferred to and State(s) or among the States, making
‘Federation of States’ for two reasons: recommendations particularly for better
1. The Indian Federation is not the result of an coordination of policy and action on these
agreement among the states like the American subjects and deliberating upon such other
Federation matters of general interest to the States which
2. The states have no right to secede from the may be referred to it by its Chairman.
federation. ● Statement 2 is incorrect: Ten members (including
● The federation is an Union because it is the chairman) shall form the quorum for a
indestructible. The country is an integral whole meeting of the council. The Constitution of India
and divided into different states only for the in Article 263, provided that an Inter-State
convenience of administration. Council may be established ,if at any time it
● Thus, Statement 1 is not correct and Statement 2 appears to the President that the public interests
and statement 3 is correct. would be served by the establishment of a
Council.
Q.3) Ans: a
Q.5) Ans: c ● Statement 3 is correct: Water is in the State List,
Exp: however the Central government has the power
● Pair 1 is correct : Cauvery Water Dispute dispute to regulate and develop inter-state rivers and
occurred between Karnataka, Tamil nadu and river valleys. The very first inter-state water
Kerala. It involves 3 states and one Union disputes tribunal was the Krishna Water Disputes
Territory i.e. Puducherry. It entailed the principle Tribunal formed in 1969. The states involved were
that the upper riparian state must obtain consent Karnataka, Andhra Pradesh and Maharashtra.
of the lower riparian state for any construction
activity. Q.7) Ans: b
● Pairs 2 and 3 are also correct : Narmada Exp:
dispute occurred between Rajasthan and ● Statement 1 is incorrect: The Inter-
Madhya pradesh and the Godavari water State Water Disputes Act empowers the Centr
dispute is between Maharashtra and Andhra al
pradesh. government to set up an ad hoc tribunal for t
o Major inter state river water disputes are : he adjudication of a
o Ravi and beas - Punjab ,Haryana ,Rajasthan dispute between two or more states in relatio
. n to the waters of an inter-
o Narmada -Madhya pradesh ,Gujrat state river or river valley.
,Maharashtra ,Rajasthan . ● Statement 2 is correct:
o Krishna - Maharashtra, Karnataka and The decision of the tribunal would be
Andhra pradesh. final and binding on the parties to the dispute
o Cauvery-Karnataka ,Kerala, Tamil nadu. . Neither the Supreme
o Godavari -Maharashtra, Karnataka,Andhra Court nor any other court is to have jurisdicti
pradesh, Madhya pradesh and Odisha. on in respect of any
water dispute which may be referred to such
Q.6) Ans: a a tribunal under this Act.
Exp:
Water sharing is one of the most contentious issues in Q.8) Ans: c
Indian federal political setup which has time and again Exp:
challenged the principle of cooperative federalism. ● Statement 1 is incorrect: Article 263 of the
● Statement 1 is incorrect: Article 262: It makes Constitution of India provides for the
two provisions w.r.t. adjudication of inter- state establishment of an Inter-State Council. The
water disputes. Council is a recommendatory body to investigate
o Parliament may by law provide for the and discuss subjects, in which some or all of the
adjudication of any dispute or complaint with states or the union government have a common
respect to the use, distribution and control of interest, recommendations for the better
waters of any inter-state river and river valley. coordination of policy and action, matters of
o Parliament may also provide that neither the general interest to the states.
Supreme Court nor any other court is to ● Statement 2 is incorrect: The inter-state council is
exercise jurisdiction in respect of any such not a permanent constitutional body for
dispute or complaint coordination between the states and union
● Statement 2 is correct: Parliament enacted two government. It can be established ‘at any time’ if
laws under these provisions: it appears to the President that the public
o River Boards Act 1956: A river board is interests would be served by the establishment of
established by the Centre on the request of the such a council.
state to advise them. ● Statement 3 is correct: The Council shall consist
o Inter-State Water Disputes Act 1956: of the prime minister who is the chairman, chief
Empowers Centre to set up an ad hoc tribunal ministers of all states who are members, chief
for the adjudication of an inter-state water ministers of union territories and administrators
dispute between two or more states; Decisions of UTs as members, and six union ministers of
of the tribunal are binding. cabinet rank in the union council of ministers
nominated by the prime minister are also by rotation, holding office for a period of one year
members. at a time.
Article 263 specifies the duties that can be ● Statement 3 is correct:
assigned to it in the following manner: The objectives (or the functions) of the zonal
● enquiring into and advising upon disputes whic councils, are as follows:
h may arise between states o To achieve an emotional integration of the
● investigating and discussing subjects in which country.
the states or the o To help in arresting the growth of acute sta
● Centre and the states have a common interest in te-consciousness, regionalism, linguism and
making particularistic trends.
recommendations upon any such subject, and o To help in removing the after-
particularly for the better coordination of policy effects of separation in some
and action on it. cases so that the process of
● Under the above provisions of Article 263, the reorganisation, integration and economic
president has established the following councils advancement may synchronise.
to make recommendations for the better o To enable the Centre and states to cooperate
coordination of policy and action in the related with each other in
subjects: social and economic matters and exchange i
o Central Council of Health and FamilyWelfare. deas and Exp:erience in order to evolve
o Central Council of LocalGovernment uniform policies.
o Four Regional Councils for Sales Tax for the
Northern, Eastern, Western and Q.10) Ans: d
SouthernZones. Exp:
The Sarkaria Commission on Centre-
Q.9) Ans: d State Relations :
Exp: ● The Sarkaria Commission on Centre-
● Statement 1 is incorrect: State Relations (1983–88) made a strong case for
The Zonal Councils are the statutory (and not the establishment of a permanent Inter-State
the constitutional) Council under Article 263 of the Constitution.
bodies. They are established by an Act of the ● It recommended that in
Parliament, that is, the order to differentiate the Inter-
States Reorganisation Act of 1956. The act div State Council from other bodies
ided the country into established under the same Article 263, it mus
five zones (Northern, Central, Eastern, Western t be called as the Inter-Governmental Council.
and Southern) and provided a zonal council for The Commission recommended that the Council
each zone. There is North-Eastern Council in should be charged with the duties laid down in
addition to the above Zonal Councils, was Article 263
created by a separate Act of Parliament, the o enquiring into and advising upon disputes w
North-Eastern Council Act of 1971. hich may arise between states;
● Statement 2 is incorrect: o investigating and discussing subjects in which
Each zonal council consists of the following m the states or the Centre and the states have a
embers common interest;and
o Home minister of the Central government o making recommendations upon any such sub
o chief ministers of all the States in the zone. ject, and particularly for the better
o Two other ministers from each state in the coordination of policy and action on it
zone.
o Administrator of each union territory in the Q.11) Ans: a
zone. Exp:
● The home minister of the Central government ● Statement 1 is correct: Public Account Committee
is the common (PAC) scrutinizes the Audit Report on
chairman of the five zonal councils. Each chief Appropriation Account and the Audit Report on
minister acts as a vice-chairman of the council Finance Account as submitted by Comptroller and
Auditor General of India (CAG). On the other Houses, the states can also have a Legislative
hand, the Committee on Public Undertaking (CPA) Council in addition to the Legislative Assembly
scrutinizes the Audit Report on Public through Article 169 of the Constitution. The
Undertaking as submitted by the CAG. PAC resolution for creation or abolition of legislative
examines the accounts showing the appropriation council must be passed by the state assembly by
of the sums granted by the House to meet the a special majority i.e., by a majority of the total
Exp:enditure, the annual Finance Accounts of the membership of the assembly and a majority of
government and, Such other accounts laid before not less than two-thirds of the members of the
the House as the Committee may think fit. assembly present and voting. At present, only six
● Statement 2 is incorrect: Both PAC and CPA do states have legislative councils. These are
submit reports to Parliament but directly and not Maharashtra, Andhra Pradesh, Telangana,
via the President. Karnataka, Bihar and Uttar Pradesh.
● Statement 3 is correct: Both PAC and CPA consist ● Statement 2 is incorrect. Parliament can pass
of 22 members of which not more than 15 belong such an act by a simple majority i.e., by a majority
to Lok Sabha and not more than 7 belong to the of the members present and voting in both the
Rajya Sabha. Further members are elected by the houses.
respective houses on the basis of proportional ● Composition of the council: Under Article 171 of
representation by means of a single transferable the Constitution, the Legislative Council of a state
vote. Thus, all parties get due representation in it. shall not have more than one-third of the total
The term of office of the members is One year. strength of the State Assembly, and not less than
40 members.
Q.12) Ans: b ● Like the Rajya Sabha, the legislative council is a
Exp: continuing chamber, that is, it is a permanent
● Statement 1 is incorrect: The State legislative body and is not subject to dissolution. The tenure
council does not participate in the election of the of a Member of the Legislative Council is six years,
president. Only the elected members of both the with one-third of the members retiring every two
Houses of Parliament, the elected members of the years.
legislative assemblies of the states and the ● Statement 3 is correct. This Act of Parliament is
elected members of the legislative assemblies of not deemed as an amendment of the Constitution
the Union Territories of Delhi and Puducherry for the purposes of Article 368. Hence, it is passed
participate in the election of the president. like ordinary legislation.
● Statement 2 is incorrect. The council has no
effective say in the ratification of a constitutional Q.14) Ans: b
amendment bill. In this respect also, the will of Exp:
the assembly prevails over that of the council. About Adjournment Motion:
● Statements 3 and 4 are correct. The council has ● Adjournment motion is introduced only in the
equal powers like that of the assembly with Lok Sabha to draw attention of the house to a
respect to; definite matter of urgent public matter. It is an
o Introduction and passage of ordinary bills extraordinary device as it interrupts the normal
o Selection of ministers, including the chief business of the House and needs the support of
minister 50 members to be admitted.
o Consideration of the reports of the ● The discussion on an adjournment motion should
constitutional bodies like the State Finance last not for less than two hours and thirty
Commission, State Public Service Commission minutes.
o Enlargement of jurisdiction of the state public ● The adjournment motion should not raise a
service commission, approval of ordinances question of privilege. It should not revive
issued by the governor. discussion on a matter that has been discussed in
the same session. It should not deal with any
Q.13) Ans: c matter that is under adjudication by court
Exp: ● It involves an element of censure against the
● Statement 1 is incorrect: India has a bicameral government and hence Rajya Sabha is not
system of legislature. Just as Parliament has two permitted to make use of this device.
Q.15) Ans: c were constituted. At present, there are eight
Exp: Parliamentary forums. They are -
About Censure Motion o Parliamentary Forum on Water Conservation
● It can be moved against any individual minister or and Management
a group of ministers or the entire council of o Parliamentary Forum on Youth
ministers. A censure means an Exp:ression of o Parliamentary Forum on Children
strong disapproval or harsh criticism. A censure o Parliamentary Forum on Population and Public
motion can be moved in the lower house of the Health
parliament or in a state assembly in India. o Parliamentary Forum on Global Warming and
● Censure motion is moved for censuring the Climate Change
council of ministers for specific policies and o Parliamentary Forum on Disaster Management
motions. No one from the council of ministers o Parliamentary Forum on Artisans and Crafts-
needs to resign if the motion is passed in the people
parliament. o Parliamentary Forum on Millenium
● Other important motions in Parliament: Development Goals.
o No-Confidence Motion: The Council of ● Statement 2 is incorrect: Speaker of the Lok
Ministers stays in office so long as it enjoys the Sabha is the ex-officio President of all the Forums
confidence of the majority of the members of except the Parliamentary Forum on Population
the Lok Sabha (Article 75). The Lok Sabha can and Public Health wherein the Chairman of Rajya
remove the ministry from office by passing a no- Sabha is the ex-officio President and the Speaker
confidence motion. The motion needs the is the ex-officio, Co-President.
support of 50 members to be approved. It can
be moved against the entire council of Q.17) Ans: d
ministers only. Exp:
o Privilege Motion: It is concerned with the About disqualifications under the Tenth Schedule:
breach of parliamentary privileges by a minister. ● The Anti-Defection Law was passed through the
It is moved by a member when he feels that a 52nd amendment in 1985 to the Constitution. It
minister has committed a breach of privilege of added the 10th Schedule to the Indian
the House or one or more of its members by Constitution with the main intent of the law being
withholding facts of a case or by giving wrong or to combat “the evil of political defections”.
distorted facts. Its purpose is to censure the ● Tenth Schedule states that a person who has been
concerned minister. elected to the office of -
o Calling Attention Motion: It is introduced in the o the Speaker or
Parliament by a member to call the attention of o the Deputy Speaker of the House of the People
a minister to a matter of urgent public or
importance and to seek an authoritative o the Deputy Chairman of the Council of States or
statement from him on that matter. It is an o the Chairman or the Deputy Chairman of the
Indian innovation in the parliamentary Legislative Council of a State or
procedure and has been in existence since o the Speaker or the Deputy Speaker of the
1954. It is mentioned in the Rules of Procedure. Legislative Assembly of a State shall not be
disqualified under this Schedule if
Q.16) Ans: d ● If he, by reason of his election to such office,
Exp: voluntarily gives up the membership of the
● Statement 1 is incorrect: Parliamentary forums political party to which he belonged immediately
are constituted for sharing of knowledge and before such election and does not, so long as he
providing a platform of discussions and dialogue continues to hold such office thereafter, re-join
between parliament members and government that political party or become a member of
officials and key Exp:erts. The Parliamentary another political party; or
Forum on Water Conservation and Management ● if he, having given up by reason of his election to
was the first to be constituted in 2005. such office his membership of the political party
Subsequently, seven more Parliamentary forums to which he belonged immediately before such
election, re-joins such political party after he transaction of the business of the Government of
ceases to hold such office. India, and for the allocation among the Ministers
of the said business”.
Q.18) Ans: b ● Statement 2 is incorrect: The total number of the
Exp: Council of Ministers cannot exceed 15%, and not
● Statement 1 is incorrect: Indian Parliamentary 10%, of the total strength of the Lok Sabha. This
Group is an autonomous body formed in the year provision was added to the Constitution by the
1949 in pursuance of a motion adopted by the 91st Amendment Act of 2003.
Constituent Assembly ● Statement 3 is incorrect. Under Article 74 of the
● Statement 2 is correct: Membership of IPG is Constitution, the question whether any, and if so
open to all members of Parliament. Former what, advice was tendered by Ministers to the
members of Parliament can also become President shall not be inquired into in any court.
associate members of the Group, but are entitled Thus, the Supreme court too cannot enquire into
to limited rights only. For example, they are not any advice given by the Council of Ministers to the
entitled to representation at meetings and President.
conferences of the IPU and the CPA and they are
also not entitled to the travel concessions Q.20) Ans: c
provided to members by certain branches of the Exp:
CPA About Cabinet Committees:
● Statement 3 is incorrect: Speaker of The Lok ● Cabinet committees are extra-constitutional
Sabha is the ex officio president of the Group. bodies, i.e., they are not mentioned in the
The Deputy Speaker of the Lok Sabha and the Constitution. However, the Rules of Business
Deputy Chairman of the Rajya Sabha are the ex provide for their establishment.
officio vice-presidents of the Group. The ● At present there are 8 functional cabinet
Secretary-general of the Lok Sabha acts as the ex committees. Committees headed by the Prime
officio Secretary-General of the Group. Minister:
● Statement 4 is correct. The aims and objectives of o Cabinet committee on Political affairs
the Group are mentioned below: o Cabinet committee on Economic affairs
o Promoting personal contacts between members o Appointment committee of the cabinet
of the Parliament of India. o Cabinet committee on Investment and growth
o Studying questions of public importance that o Cabinet committee on Security
are likely to come up before the Parliament; o Cabinet committee on Employment and Skill
arrange seminars, discussions and orientation Development
courses; and bring out publications for the ● Committees headed by the home minister:
dissemination of information to the members of Cabinet committee on Accommodation
the Group. ● Committee Headed by the defence minister:
o Arranging lectures on political, defense, Cabinet committee on Parliamentary affairs
economic, social and educational problems by
the members of the Parliament and Q.21) Ans: b
distinguished persons. Exp:
o Arranging visits to foreign countries with a view ● Statement 1 is incorrect: The representatives of
to develop contacts with members of other each community were to be elected by members
parliaments. of that community in the provincial legislative
assembly and voting was to be by the method of
Q.19) Ans: d proportional representation by means of single
Exp: transferable vote.The representatives of princely
● Statement 1 is incorrect: Under the Constitution, states were to be nominated by the heads of the
it is the President not the Prime Minister who princely states. It is thus clear that the
makes the rules for the allocation of the business Constituent Assembly was to be a partly elected
of the Government of India among the Ministers. and partly nominated body. Moreover, the
According to Article 77 of the Constitution, “The members were to be indirectly elected by the
President shall make the rules for the convenient
members of the provincial assemblies, who ● The retiring members are eligible for re-election
themselves were elected on a limited franchise. and renomination any number of times.
● Statement 2 is correct: Although the Constituent ● The Constitution has not fixed the term of office
Assembly was not directly elected by the people of members of
of India on the basis of adult franchise, the the Rajya Sabha and left it to the Parliament.
Assembly comprised representatives of all ● Accordingly, the
sections of Indian Society—Hindus, Muslims, Parliament in the Representation of the People
Sikhs, Parsis, Anglo–Indians, Indian Christians, Act (1951)
SCs, STs including women of all these sections. provided that the term of office of a member
The Assembly included all important of the Rajya Sabha shall be six years. The act
personalities of India at that time, with the also empowered the president of India to curtail
exception of Mahatma Gandhi. the term of members chosen in the first Rajya
Sabha. In
Q.22) Ans: c the first batch, it was decided by lottery as to
Exp: who should retire.
For the purpose of holding direct elections to the Lok ● Further, the act also authorised the
Sabha, each President to make provisions
state is divided into territorial constituencies. In th to govern the order of retirement of the mem
is respect, the Constitution makes the following two bers of the Rajya Sabha.
provisions:
● Statement 1 is correct: Each state is allotted a Q.24) Ans: a
number of seats in the Lok Sabha in Exp:
such a manner that the ratio between that nu About No-Confidence Motion
mber and its population is the same for all ● Art. 75: Council of Ministers shall be collectively
states. This provision does not apply to a state responsible to the Lok Sabha. This principle is the
having a population of less than six millions. bedrock of parliamentary democracy. Ministry
Constitution ensures that there is uniformity o stays in office till they enjoy confidence of the
f representation between the different states majority of the members of the Lok Sabha
● Statement 2 is correct: ● No-Confidence Motion is not mentioned in the
Each state is divided into territorial constituen Constitution, is moved under Rule 198 of rules of
cies in such a procedure and can be moved only in Lok Sabha.
manner that the ratio between the population ● Needs support of 50 members, no need to state
of each the reasons for its adoption.
constituency and the number of seats allotted ● The governments formed with wafer-
to it is the same throughout the state. The thin majority have been called upon by the
Constitution ensures that there is uniformity President to prove their majority on the floor of
between the different constituencies in the same the House. The government of the day,
state. sometimes, on
its own, seeks to prove its majority by moving
Q.23) Ans: d a motion of
Exp: confidence and winning the confidence of the
About the Rajya Sabha House. If the
● Duration of confidence motion is negative, it results in the
The Rajya Sabha (first constituted in 1952) is fall of the government.
a continuing ● No-Confidence Motion is moved only against the
chamber, that is, it is a permanent body and entire CoM (Not against individual/ group of
not subject to dissolution. However, one-third of ministers) and if passed, the CoM must resign
its members retire every second year. from office.
● Their seats are filled up by fresh elections and
presidential Q.25) Ans: d
nominations at the beginning of every third ye Exp:
ar.
● Statement 1 is incorrect: According to the subordinate judiciary are laid down by the stat
Constitution, the Speaker of the last Lok Sabha es. Below the District and Sessions Court stands
vacates his office immediately before the first the Court of
meeting of the newly-elected Lok Sabha. Speaker Subordinate Judge on the civil side and the Co
pro-Tem institution facilitates the transition of the urt of Chief Judicial
institution from old ones to newly elected Magistrate on the criminal side. The subordina
members. He also enables the House to elect the te judge exercises
new Speaker. Therefore, the President appoints a unlimited pecuniary jurisdiction over civil suits
member of the Lok Sabha as the Speaker Pro- ● Statement 2 is incorrect: The chief judicial
Tem. Usually, the senior most member is selected magistrate decides criminal cases which are pu
for this. nishable with
● Statement 2 is incorrect: The President himself imprisonment for a term up to seven years.
administers the oath to the Speaker Pro Tem. He At the lowest level, on the civil side, is the C
presided over the first sitting of the newly-elected ourt of Munsiff and on
Lok Sabha. Main onus of Speaker Pro-Tem is to the criminal side, is the Court of Judicial Magi
administer oaths to the new members. The strate. The munsiff possesses
Speaker Pro Tem has all the powers of the limited jurisdiction and decides civil cases of
Speaker. This office is a temporary office, when small
the new Speaker is elected by the House, the
office of the Speaker Pro Tem ceases to exist. Q.28) Ans: d
Exp:
Q.26) Ans: b Statement 4 is incorrect:
Exp: ● The NALSA lays down policies, principles, guide
● Statement 1 is incorrect: The lines and frames
state judiciary consists of a high court and a effective and economical schemes for the Stat
hierarchy of subordinate courts, also e Legal Services
known as lower courts. Authorities to implement the Legal Services Pr
Articles 233 to 237 in Part VI of the Constituti ogrammes throughout the country.
on make the following Primarily, the State Legal Services Authorities,
provisions to regulate the organization of subo District Legal
rdinate courts and to Services Authorities, Taluk Legal Services Com
ensure their independence from the executive. mittees, etc. have been
The appointment, posting and promotion of di asked to discharge the following main function
strict judges in a state are s on a regular basis:
made by the governor of the state in consulta o To provide free and competent legal
tion with the high court. services to the eligible persons.
● Statement 2 is correct: o To
Appointment of persons (other than district ju organise Lok Adalats for amicable settlement
dges) to the judicial of disputes.
service of a state are made by the governor o o To organise legal awareness camps
f the state after in the rural areas.
consultation with the State Public Service Com ● Act also provide for the free legal services which
mission and the high court. include:
The control over posting, promotion and o Payment of court fee, process
leave of persons belonging to the judicial fees and all other charges payable
service of a state is vested in the high court. or incurred in connection with any legal
proceedings.
Q.27) Ans: d o Providing service of lawyers in legal
Exp: proceedings.
● Statement 1 is incorrect: o Obtaining and supply of certified copies of o
The organizational structure, jurisdiction and n rders and other documents in legal
omenclature of the proceedings.
o Preparation of appeal, paper book including o In the original Side the District Judge has the
printing and translation of documents in legal jurisdiction of unlimited valuation .
proceedings. o In the appellate side the District Judge has the
● There is provision for free legal services. Jurisdiction of limited valuation , he can hear
The persons eligible for getting free legal the appeals from the order of Civil Judge (SD)
services include: and Civil Judge(JD) of the cases having valuation
o Women and children up to Rs.5 lacs.
o Members of SC/ST o The District Judge has the revisional Jurisdiction.
o Industrial Workmen o The District Judge can transfer the cases to the
o Victims Additional District Judge.
of mass disaster, violence, flood, drought, ea o The District Judge can assign the administrative
rthquake industrial disaster power among the Judicial officer
o Disabled persons
o Persons in custody Q.31) Ans: d
o Persons whose annual Exp:
income does not exceed ₹1 lakh (in the ● Statement 1 is correct: The Law Commission of
Supreme Court Legal Services Committee the India, in its 114 th Report, had suggested
limit ₹1,25,000/-) establishment of Gram Nyayalayas for providing
o Victims of trafficking in human beings affordable and quick access to justice to the
orbegar. citizens at their doorsteps. The Gram Nyayalayas
Act came into force with effect from 02nd
Q.29) Ans: d October, 2009. It is a Statutory and Quasi-judicial
Exp: body established by Gram Nyayalayas Act 2008.
● Statement 1 is incorrect: The Act extends to the whole of India, except to
The organizational structure, jurisdiction and n the States of Nagaland, Arunachal Pradesh, Sikkim
omenclature of the and to the tribal areas specified in the Sixth
subordinate judiciary are laid down by the stat Schedule to the Constitution of India within the
es. Below the District and Sessions Court stands States of Assam, Meghalaya, Tripura and
the Court of Mizoram, respectively.
Subordinate Judge on the civil side and the Co ● Statement 2 is correct: Gram Nyayalayas are
urt of Chief Judicial aimed at providing inExp:ensive justice to people
Magistrate on the criminal side. The subordina in rural areas at their doorsteps; Gram Nyayalayas
te judge exercises are to be established for every Panchayat at
unlimited pecuniary jurisdiction over civil suits intermediate level or a group of contiguous
● Statement 2 is incorrect: The chief judicial Panchayats at intermediate level or for a group of
magistrate decides criminal cases which are pu contiguous Gram Panchayat
nishable with ● Statement 3 is correct: It ensures the provision
imprisonment for a term up to seven years. under 39A (Added by 42nd CAA 1976) in Directive
At the lowest level, on the civil side, is the C Principle of State Policy (Part IV) of Indian
ourt of Munsiff and on constitution directs the State to ensure equal
the criminal side, is the Court of Judicial Magi justice on a basis of equal opportunity and shall
strate. The munsiff possesses provide free legal aid”.
limited jurisdiction and decides civil cases of
small Q.32) Ans: d
Exp:
Q.30) Ans: d ● Statement 1 is incorrect: Gram Nyayalaya Provide
Exp: access to justice to the most marginalized sections
● Civil Jurisdiction of District Judge:- of the society. It reduces barriers to access in
o The principal Court of Civil Jurisdiction is the terms of distance, time & associated costs w.r.t.
Court of District Judge at the District level. justice. There are Alternate Dispute redressal
o It has both original as well as appellate mechanisms- Arbitration-conciliation-mediation It
Jurisdiction.
ensures cheap, affordable and effective justice to than its headquarters. The State Government
downtrodden strata of society. shall appoint a presiding officer called Nyay
● Statement 2 is incorrect: It is established for Adhikari for every Gram Nyayalaya in consultation
every Panchayat at intermediate level or a group with the High Court
of contiguous Panchayats at intermediate level in ● Statement 2 is incorrect: The Gram Nyayalaya
a district The State Government, in consultation shall be the court of Judicial Magistrate of the first
with the High Court, notified the boundaries of class. The salary and other allowances along with
the area under the jurisdiction of a Gram the other terms and conditions of service shall be
Nyayalaya. It can also alter such limits at any time. on the same lines as well. Representation shall be
It can hold mobile courts in villages falling under given to the members of the SC, ST, women and
its jurisdiction and the State Government shall others.
extend all required facilities.
Q.35) Ans: d
Q.33) Ans: c Exp:
Exp: ● Statement 1 is incorrect: Gram Nyayalaya is not
Jurisdiction Power and Authority: bound by the Indian Evidence Act, 1872 but is
● The Gram Nyayalaya shall be a mobile court and guided by the principles of natural justice and is
shall exercise the powers of both Criminal and subject to any rule made by the High Court. The
Civil Courts. nature of jurisdiction specified in the Act can be
● The Gram Nyayalaya shall try criminal cases, civil amended by both the central and state
suits, claims or disputes which are specified in the governments.
First Schedule and the Second Schedule to the ● Statement 2 is incorrect: The Gram Nyayalaya
Act. shall follow summary procedure in criminal
● The Gram Nyayalaya shall exercise the powers of trials.The judgement and order passed by the
a Civil Court with certain modifications and shall Gram Nyayalaya shall be deemed to be a decree
follow the special procedure as provided in the and to avoid delay in its execution. Gram
Act. Nyayalaya shall follow summary procedure for its
● The Gram Nyayalaya shall try to settle the execution.
disputes as far as possible by bringing about o Meaning of Summary Procedure– It is a legal
conciliation between the parties and for this procedure used for enforcing a right that takes
purpose. It shall make use of the conciliators to be effect faster and more efficiently than ordinary
appointed for this purpose. methods.
● Gram Nyayalaya can try criminal cases, civil suits, ● Statement 3 is incorrect: For criminal
claims or disputes which are specified in the Act. cases appeal shall be taken to the Court of
For e.g. Offences not punishable with death, Session and for civil cases appeal shall be taken to
imprisonment for life or imprisonment for a term District court. Appeals in both cases have to be
exceeding two years. heard and disposed of within six months.
● Gram Nyayalaya is not bound by the Indian
Evidence Act, 1872 but is guided by the principles Q.36) Ans: a
of natural justice and is subject to any rule made Exp:
by the HIGH COURT. About Lok Adalat
● The nature of jurisdiction specified in the Act can Lok Adalat is an alternative dispute resolution
be amended by both the central and state mechanism. The cases that are pending in courts or
governments. are at the pre-litigation stage are settled in Lok Adalat.
Permanent Lok Adalat is a permanent body
Q.34) Ans: d constituted to settle cases relating to Public Utility
Exp: Services like transport, postal, telegraph, etc. A
● Statement 1 is incorrect: The seat of the Gram Chairman and two members are appointed for
Nyayalayas shall be located at the headquarters of settling disputes through conciliation.
the intermediate Panchayat. The Nyayadhikari ● Statement 1 is correct: The Lok Adalat has
shall periodically visit villages and may hear the jurisdiction to attend any case that is pending
parties and dispose of the cases at the place other before a court of law, and cases that are likely to
be filed before the court of law. The Permanent through conciliation and compromise. In case of
Lok Adalat has jurisdiction to attend cases only failure of the parties to arrive at a settlement, the
relating to Public Utility Services such as case reverts back to the court from where it is
transport, postal, telegraph sanitation, banking, referred.
education, dispensary, power, light, water, ● A Lok Adalat was established by the government
insurance, housing, and real estate services. The to settle the disputes by mutual consent. It is
pecuniary jurisdiction of Permanent Lok Adalats presided over by a sitting or retired judicial officer
has increased from Rs. 10 lacs to Rs. One crore. as the chairman with a lawyer and a social worker.
● Statement 2 is incorrect: The disputes before Lok ● The decision of a lok adalat is binding on the
Adalats are settled by conciliators who don’t have parties and the order can be executed like any
any judicial role. Thus, the conciliators can only decree passed by a court. There is no right of
persuade the parties to settle the dispute, they do appeal against the decision of a lok adalat. A lok
not have the authority to make a decision without adalat has jurisdiction to try all matters except
the consent of the parties. This is a major those relating to non-compoundable offences.
drawback of Lok Adalats. The whole procedure
becomes futile if one of the parties is not willing Q.38) Ans: c
to reach a settlement, even if the conciliators are Exp:
of the opinion that the dispute is capable of being In each district of India there are various types of
resolved through conciliation. subordinate or lower courts. They are civil courts,
● Statement 3 is incorrect: The Legal Services criminal courts and revenue courts. These Courts hear
Authorities Act states that an award made by a civil cases, criminal cases and revenue cases,
Lok Adalat is deemed to be a decree of a civil respectively.
court and no appeal shall lie against such orders. ● Statement 1 is correct: Civil cases pertain to
Thus, the parties cannot file an appeal against the disputes between two or more persons regarding
decision of the Lok Adalats, although the parties’ property, breach of agreement or contract,
have the discretion to approach the appropriate divorce or landlord – tenant disputes. Civil Courts
court and file a fresh civil suit if they are aggrieved settle these disputes. They do not award any
by the decision of the Lok Adalat. By doing so, the punishment as violation of law is not involved in
parties exercise their right to litigate. civil cases.
● Statement 2 is correct: Criminal cases relate to
Q.37) Ans: b violation of laws. These cases involve theft,
Exp: dacoity, rape, pick- pocketing, physical assault,
About the Lok Adalat murder, etc. These cases are filed in the lower
● Lok Adalat is an alternative dispute resolution court by the police, on behalf of the state, against
mechanism. The cases that are pending in courts the accused. In such cases the accused, if found
or are at the pre-litigation stage are settled in Lok guilty, is awarded punishment like fine,
Adalat. The cases are settled through conciliation, imprisonment or even death sentence.
which is why Lok Adalats are one of the most Revenue cases relate to land revenue on agricultural
important components of the ADR system land in the district.
operating in India.
● Lok adalats remain the major area of activity of Q.39) Ans: c
the LSAA. Section 2(1) (d) of the LSAA defines ‘lok Exp:
adalat’ to mean a lok adalat organised under ● Statement 1 is incorrect: The Court of the District
Chapter VI of the Act. Lok adalats are organised by Judge is the highest civil court in a district to deal
legal services authorities at the state, district and with civil cases. Very often the same court is
taluka levels in each state. called the Court of District and Sessions Judge,
● A case that is pending in a court can be referred when it deals with both civil and criminal cases at
to a lok adalat by any one of the parties to the the district level. The judge of this court is
litigation. appointed by the Governor of the State.
● There is no court fee and any fee paid in the court ● Statement 2 is correct: Below the Court of District
is refunded provided the dispute is settled in the Judge, there may be one or more courts of sub
lok adalat. Cases in a lok adalat are settled judges in the district. Separate family courts,
which are equal to sub judge, have been o conducting legal awareness camps in rural as
established in districts to exclusively hear cases of well as urban areas;
family disputes, like divorce, custody of children, o through print media like newspapers, preparing
etc. Below them there are courts of munsifs and and publishing legal-aid literature;
small causes courts which decide cases involving o arranging workshops on legal topics for various
petty amounts. No appeal can be made against functionaries of the legal systems such as
the decisions of the small causes courts. All these lawyers, students, Non-Governmental
courts hear and settle civil disputes. organizations and Government agencies,
● Statement 3 is incorrect: The Court of the District o carrying out field surveys,
Judge (called the District Courts) hears not only o creating awareness about the law of the land
appeals against the decisions of the courts of sub amongst the general public by utilizing digital
judges, but also some of the cases begin directly mass media like Radio, Television, Internet,
in the Court of the District Judge itself. Appeals social media and the like etc.
against the decisions of this court may be heard o NALSA also facilitates the beneficiaries of
by the High Court of the State. Civil Courts deal various government schemes, policies to avail
with cases pertaining to disputes between two or the benefits. It facilitates resolution of disputes
more persons regarding property, divorce, through alternative dispute resolution
contract, and breach of agreement or landlord – mechanisms such as Lok Adalat, Mediation, etc.
tenant disputes.
Q.41) Ans: d
Q.40) Ans: c Exp:
Exp: ● Option A is incorrect: The council can only
The National Legal Services Authority (NALSA) has discuss the budget but cannot vote on the
been constituted under the Legal Services Authorities demands for grants. It is the exclusive privilege of
Act, 1987 to provide free Legal Services to the weaker the assembly.
sections of the society and to organize Lok Adalats for ● Option B is incorrect: The council cannot remove
amicable settlement of disputes. the council of ministers by passing a no-
● Statement 1 is incorrect: Legal Services includes confidence motion. This is because the council of
providing Free Legal Aid to those weaker sections ministers is collectively responsible only to the
of the society who fall within the purview of assembly. But, the council can discuss and
Section 12 of the Legal Services Authority Act, criticize the policies and activities of the
1987. Free legal aid is not confined to cases Government.
before the subordinate Courts. Legal Aid is ● Option C is incorrect: The council can be
provided to the needy from the lowest Court to abolished by the Parliament on the
the Supreme Court of India. Legal Aid Counsel recommendation of the assembly. The very
represent such needy persons before the lower existence of the council depends on the will of the
Courts, High Courts and also before the Supreme assembly.
Court of India. ● Option D is correct: The council does not
● Statement 2 is correct: According to Regulation participate in the election of the president of
7(5) of the National Legal Services Authority (Free India and representatives of the state in the
and Competent Legal Services) Regulations 2010, Rajya Sabha. The council has no say in the
the application for legal services will be ratification of a constitutional amendment bill. In
scrutinized by the Member-Secretary or the this respect also, the will of the assembly prevails
Secretary and if any individual is aggrieved by the over that of the council.
decision made upon their application, he/she has
Extra Edge by Only IAS
an option to appeal to the Executive Chairman or
● India follows a bicameral system at both
Chairman of the Legal Services Institution and the
the center and state level. Under this
decision resulting from the appeal would be a
system, the state's legislature is divided
final one.
into two parts - Legislative Assembly or
● Statement 3 is incorrect: Besides legal aid, NALSA
Vidhan Sabha and Legislative Council or
also spreads legal literacy and awareness by
Vidhan Parishad.
for presidential assent. He cannot return the bill
● The Legislative Council is the upper house
for reconsideration of the state legislature.
of the state. Its institution is outlined in
Normally, the governor gives his assent to a
Article 169 of the Constitution of India.
money bill as it is introduced in the state
● The members of the Council are either
legislature with his prior permission.
nominated by the Governor of the state or
● Option B is correct: The president can either give
are indirectly elected.
his assent to the bill or withhold his assent to the
bill. He cannot return the bill for reconsideration
Q.42) Ans: a of the Houses.
Exp: ● Option C is correct: The decision of the Speaker
● Statement 1 is incorrect: A Money Bill cannot be of assembly whether a bill is a money bill or not
introduced in the legislative council. It can be is final. His decision cannot be questioned in any
introduced only in the legislative assembly. court of law or in either the House of State
● Statement 2 is incorrect: The speaker of the Legislature or even the governor. When the bill is
house decides whether a bill is a money bill or sent to the Legislative Council for review and
not and his decision on this question is final. submitted to the Governor for approval, the
Money Bill can be introduced in the legislative speaker confirms it as a Money bill.
assembly only on the recommendation of the ● Option D is incorrect: After a Money Bill is passed
governor. by the legislative assembly, it is transmitted to the
● Statement 3 is correct: After a Money Bill is legislative council for its consideration.
passed by the legislative assembly, it is o The Legislative Council cannot reject or amend
transmitted to the legislative council for its a money bill. It can only make the
consideration. recommendations. It must return the bill to the
o The Legislative Council cannot reject or amend assembly within 14 days, with or without
a money bill. It can only make the recommendations.
recommendations. It must return the bill to the o The Legislative assembly can either
assembly within 14 days, with or without accept/reject all or any of the
recommendations. recommendations of the Legislative Council.
o The Legislative assembly can either o If the Legislative assembly accepts any
accept/reject all or any of the recommendation, the bill is then considered to
recommendations of the Legislative Council. have been passed by both the Houses.
o If the Legislative assembly accepts any o If the Legislative assembly does not accept any
recommendation, the bill is considered to have recommendation, the bill is then considered to
been passed by both the Houses. be passed by both the Houses in the form
o If the Legislative assembly does not accept any originally approved by the Legislative
recommendation, the bill is then considered to assembly.
be passed by both the Houses in the form o There is no provision of holding a joint sitting
originally approved by the Legislative in case of disagreement between the two
assembly. houses.
o There is no provision of holding a joint sitting in o If the Legislative Council fails to return the bill
case of disagreement between the two houses. to the Legislative assembly within 14 days, the
o If the Legislative Council fails to return the bill bill is deemed to have been accepted by both
to the Legislative assembly within 14 days, the the Houses in the form originally approved by
bill is considered to have been accepted by the Legislative assembly.
both the Houses in the form originally
approved by the Legislative assembly. Q.44) Ans: d
Exp:
Q.43) Ans: d ● Statement 1 is correct: An ordinary bill can
Exp: originate in either House of the state legislature.
● Option A is correct: When a Money Bill is It can be introduced by a minister or any other
presented to the governor, he may either give member. The bill passes through 3 stages in the
his assent, withhold his assent or reserve the bill
originating House: First reading, Second reading, o He may withhold his assent to the bill: the bill
Third reading. ends and does not become an Act.
Once the bill is passed by the originating House, it o He may return the bill for reconsideration of
is transferred to the second House for the House or Houses: It becomes obligatory for
consideration and passage. the Governor to give his assent to the bill, if it
● Statement 2 is correct: Here also, the bill passes is passed by both the Houses again (with or
through all the 3 stages, that is, first reading, without amendments) and presented to him.
second reading and third reading. o He may reserve the bill for the consideration of
o Case 1: Council passes the bill without the President.
amendments or Assembly accepts the ● Option D is correct: Case 1: Council passes the
amendments suggested by the council: The bill bill without amendments or Assembly accepts
is considered to be passed by both the Houses the amendments suggested by the council: The
and is sent to the governor for his assent. bill is considered to be passed by both the Houses
o Case 2: Assembly rejects the amendments and is sent to the governor for his assent.
suggested by the council or Council rejects the o Case 2: Assembly rejects the amendments
bill altogether or Council does not take any suggested by the council or Council rejects the
action for 3 months: Assembly may pass the bill bill altogether or Council does not take any
again and transmit the same to the council. action for 3 months: Assembly may pass the bill
o Case 3: Council rejects the bill again or Passes again and transmit the same to the council.
the bill with amendments which are not o Case 3: Council rejects the bill again or Passes
acceptable to the assembly or Council does not the bill with amendments which are not
pass the bill within 1 month: The bill is acceptable to the assembly or Council does not
considered adopted by both houses in the form pass the bill within 1 month: The bill is
in which it was approved a second time by the considered adopted by both houses in the form
assembly. Therefore, the ultimate power of in which it was approved a second time by the
passing an ordinary bill resides with the assembly.
assembly.
● Statement 3 is correct: The Council can retain or Q.46) Ans: d
postpone the bill for a maximum of four months: Exp:
three months in the first instance and one month ● Statement 1 is incorrect: Members of the state
in the second instance. The Constitution does not legislature have freedom of speech. In court
provide a mechanism for both houses to meet proceedings, no member can be held liable for
together to resolve differences of opinion his declaration or vote in Parliament or its
between the two houses on a bill. committees. During the meeting of the state
legislature, they can refuse to testify in pending
Q.45) Ans: c court cases and appear in court as a witness.
Exp: ● Statement 2 is incorrect: Members cannot be
● Option A is correct: If a bill that originates from arrested during the session of state legislature
the council and has been sent to the assembly is and 40 days before the beginning and 40 days
rejected by the assembly, the bill ends and dies. after the end of a session (only in civil cases)
Therefore, the council has received much less
importance, position and authority than the Rajya Q.47) Ans: d
Sabha in the Center. Exp:
● Option B is correct but c is incorrect: Every bill, ● Option A is incorrect: Collective Privileges are the
after it is passed by the assembly or by both the privileges enjoyed by each House of Parliament
Houses is presented to the governor for his collectively. They are:
assent. There are four alternatives before the o Right to publish its reports, debates and
governor: proceedings and also prohibit others from
o He may give his assent to the bill: the bill publishing the same. It can punish members as
becomes an act and is placed on the Statute well as outsiders for breach of its privileges or
Book its contempt.
o It has the right to receive immediate casting vote in the case of equality of votes in
information of the arrest, detention, any matter.
conviction, imprisonment and release of a ● Statement 3 is incorrect: He decides the
member. questions of disqualification of a member of the
o It can institute inquiries and order the assembly, arising on the ground of defection
attendance of witnesses and send for relevant (Tenth Schedule). This decision is subject to
papers and records. judicial review. The Speaker is the principal
o It can make rules to regulate its own procedure spokesman of the House. He represents its
and the conduct of its business and to arbitrate collective voice and is its sole representative to
such matters. the world. The Speaker holds the Office from the
● Option B is incorrect: The courts are prohibited date of his election till immediately before the
to inquire into the proceedings of a House or its first meeting of the Legislative Assembly after the
committees. dissolution of the one to which he was elected,
● Option C is incorrect: Press does not require prior unless he ceases to be a member by any of the
permission of the House to publish true reports reasons specified in Article 179 of the Indian
of parliamentary proceedings but it cannot do so Constitution.
in the case of a secret sitting of the House. The
house can exclude strangers from secret sittings Q.50) Ans: a
to discuss some important matters. Exp:
● Option D is correct: No person ● Statement 1 is correct: The Office of the Speaker
(member/outsider) can be arrested, and no legal of Legislative Assembly is a constitutional one.
process (civil or criminal) can be served within Under Article 178 of the Indian Constitution every
the boundaries of the House without the Legislative Assembly of a State shall choose one of
permission of the Speaker/Chairman. its Members as Speaker. A speaker is responsible
for presiding assembly debates and maintains
Q.48) Ans: b order and discipline of the house during legislative
Exp: sessions.
● Statement 1 is incorrect: Quorum is the ● Statement 2 is correct: If the Speaker certifies a
minimum number of members required to be Bill as a Money Bill, his decision is final. Under
present in the House before it can conduct any the Constitution, the Speaker exercises only a
business. It is 1/10th of the total number of casting vote in the case of equality of votes in
members in each House including the presiding any matter.
officer. ● Statement 3 is incorrect: He decides the
● Statement 2 is correct: If there is no quorum questions of disqualification of a member of the
during a meeting of the House, it is the duty of assembly, arising on the ground of defection
the presiding officer either to adjourn the House (Tenth Schedule). This decision is subject to
or to suspend the meeting until there is a judicial review. The Speaker is the principal
quorum. spokesman of the House. He represents its
collective voice and is its sole representative to
Q.49) Ans: a the world. The Speaker holds the Office from the
Exp: date of his election till immediately before the
● Statement 1 is correct: The Office of the Speaker first meeting of the Legislative Assembly after the
of Legislative Assembly is a constitutional one. dissolution of the one to which he was elected,
Under Article 178 of the Indian Constitution every unless he ceases to be a member by any of the
Legislative Assembly of a State shall choose one of reasons specified in Article 179 of the Indian
its Members as Speaker. A speaker is responsible Constitution.
for presiding assembly debates and maintains
order and discipline of the house during legislative Q.51) Ans: d
sessions. Exp:
● Statement 2 is correct: If the Speaker certifies a ● Statement 1 is incorrect: Constitution of India
Bill as a Money Bill, his decision is final. Under contain the rule of 'immunity from mutual
the Constitution, the Speaker exercises only a
taxation' and makes the following provisions in ● The exclusive power to levy taxes on subjects
this regard: enumerated in the Union List (which are 15 in
● The Center's property is exempted from taxation number) are vested with Parliament.
imposed by state or any authority within a state ● The power to levy taxes on subjects enumerated
like municipalities, district boards, panchayats and in the Concurrent List (which are 3 in number) are
so on. vested with both the Parliament and the state
o The term 'property' includes lands, buildings, legislature.
chattels, shares, debts, everything that has a ● Statement 2 is incorrect: The state legislature has
money value, and every kind of property- exclusive power to levy taxes on subjects
movable or immovable and tangible or enumerated in the State List (which are 20 in
intangible. number). Parliament is empowered to impose
o Moreover, the property may be used for residuary taxes (that is, the power to impose
sovereign (like armed forces) or commercial taxes not enumerated in any of the three lists).
purposes. Under this provision, the Parliament has imposed
● statement 2 is incorrect: The Parliament (not gift tax, wealth tax and Exp:enditure tax. The
President) is empowered to remove the ban Constitution also makes a distinction between
which restricts property of the center from the power to levy and collect a tax and the power
taxation. to appropriate the proceeds of the tax so levied
● The corporations or the companies which were and collected. For example, the income-tax is
formed by the Central government are not levied and collected by the Center but its
immune from state taxation or local taxation. proceeds are distributed between the Center and
This is because a corporation or a company is a the states.
separate legal entity.
● The state's property and income of a state is Q.53) Ans: a
exempted from Central taxation. Such income Exp:
may be derived from sovereign functions or ● Statement 1 is correct: The distribution of
commercial functions.(But the Center can tax the legislative powers between the Center and the
commercial operations of a state (if Parliament so states is rigid. Consequently, the legislative
provides. However, the Parliament can declare powers of state cannot be delegated to the
that any particular trade or business as incidental states and a single state cannot request the
to the ordinary functions of the government and Parliament to make a law on a state subject. The
it would then not be taxable. distribution of executive power in general follows
o However, the property and income of local the distribution of legislative powers. But,
authorities situated within a state are not sometimes conflicts arise between center and
exempted from the Central taxation. states due to rigid division in the executive
o Similarly, the center can impose taxes on the sphere. Hence, inter-government delegation is
property or income of corporations and provided by the Constitution of executive
companies owned by a state. functions in order to mitigate rigidity and avoid a
o In an advisory opinion in 1963, the Supreme situation of deadlock.
court held that the immunity granted to a state o The President, with the consent of the state
in respect of Central taxation does not extend government, entrusts to that government any
to the duties of customs or duties of excise. In of the executive functions of the Center.
other words, the Center can impose customs o Conversely, the governor of a state with the
duty on goods imported or Exp:orted by a state, consent of the Central government, entrusts to
or an excise duty on goods produced or that government any of the executive functions
manufactured by a state. of the state. This mutual delegation of
administrative functions may be conditional or
Q.52) Ans: d unconditional.
Exp: ● Statement 2 is incorrect: The Constitution also
● Statement 1 is incorrect: The Constitution has makes a provision for the entrustment of the
divided the taxing powers between the Center executive functions of the Center to a state
and the states in the following way: without the consent of that state. But, the
delegation is by the Parliament and not by the c) The former provision lays down a general
president. obligation upon the state, but the latter
o Thus, a law made by the Parliament on a provision imposes a specific obligation on the
subject of the Union List can confer powers and state not to hamper the executive power of the
impose duties on a state, or authorize the Center.
conferring of powers and imposition of duties ● In both the cases, the executive power of the
by the Center upon a state (irrespective of the Center extends to giving of such directions to the
consent of the state concerned). Notably, the state as are necessary for the purpose. The
same thing cannot be done by the state purpose behind these directions of the Center is
legislature. coercive in nature.
It means the mutual delegation of functions ● Thus, Article 365 says that whenever any state
between the Center and the state can take has failed to comply with (or to give effect to)
place either under an agreement or by any directions given by the Center, it will be
legislation. While both the methods can be lawful for the President to hold that a situation
used by the centre, however a state can use has arisen in which the government of the state
only the first method. cannot be carried on in accordance with the
provisions of the Constitution. It means that, in
Q.54) Ans: b such a situation, the President's rule can be
Exp: imposed in the state under Article 356.
● Statement 1 is incorrect: The Constitution
contains the provisions which enable the Center Q.55) Ans: b
to exercise control over the state administration: Exp:
o Article 355 (not 365) imposes two duties on the ● Statement 1 is incorrect: The Constitution of India
Center: (a) to protect every state against provides for grants-in-aid to the states from the
external aggression and internal disturbance; Central resources.
and (b) to ensure that the government of every o Article 275 (not 282) empowers the Parliament
state is carried on in accordance with the to make grants to the states which are in need
provisions of the Constitution. of financial assistance and not to every state.
o The governor of a state is appointed by the ● Statement 2 is correct: The Finance Commission
president. He holds office during the pleasure recommends the statutory grants under Article
of the President. Apart from being the 275 (both general and specific) which are given to
Constitutional head of the state, the governor the states. On the other hand, Article 282
also acts as an agent of the Center in the state. empowers both the Center and the states to
The periodical reports about the administrative make any grants for any public purpose, even if it
affairs of the state are submitted by the is not within their respective legislative
governor to the Center. competence. Under this provision, the Center
o Though the governor of the state appoints makes grants to the states.
state election commissioner but he can be
removed only by the President. Q.56) Ans: a
● Statement 2 is correct: Two restrictions have Exp:
been placed by the Constitution on the executive ● Statement 1 is correct: In the US, only the powers
power of the states in order to give ample scope of the Federal Government are enumerated in the
to the Center for exercising its executive power in Constitution and the residuary powers are left to
an unrestricted manner. the states. The Australian Constitution followed
● Thus, the executive power of every state is to be the American pattern of single enumeration of
exercised in such a way powers. In Canada, on the other hand, there is a
a) that ensure compliance with the laws made by double enumeration—Federal and Provincial, and
the Parliament and any existing law which apply the residuary powers are vested in the Centre.
in the state o The Constitution provides for a three-fold
b) as not to impede or prejudice the exercise of distribution of legislative subjects between the
executive power of the Center in the state. Centre and the states, viz., List-I (the Union
List), List-II (the State List) and List-III (the List under the following five extraordinary
Concurrent List) in the Seventh Schedule. circumstances:
● Statement 2 is correct: The Government of India o When Rajya Sabha Passes a Resolution
(GoI) Act of 1935 provided for a three-fold o During a National Emergency
enumeration, viz., federal, provincial and o When States Make a Request
concurrent. The present Constitution follows the o To Implement International Agreements
scheme of this act but with one difference, that is, o During President’s Rule
under this act, the residuary powers were given ● If the Rajya Sabha declares that it is necessary in
neither to the federal legislature nor to the the national interest that Parliament should make
provincial legislature but to the governor-general laws on a matter in the State List, then the
of India. In this respect, India follows the Parliament becomes competent to make laws on
Canadian precedent. that matter. Such a resolution must be supported
● Statement 3 is incorrect: The power to make laws by two-thirds of the members present and
with respect to residuary subjects (i.e., matters voting. The resolution remains in force for one
which are not enumerated in any of the three year; it can be renewed any number of times but
lists) are vested in the Parliament. This residuary not exceeding one year at a time.
power of legislation includes the power to levy ● The Parliament acquires the power to legislate
residuary taxes. with respect to matters in the State List, while a
proclamation of national emergency is in
Q.57) Ans: c operation. The laws become inoperative on the
Exp: Exp:iration of six months after the emergency has
● Statement 1 is correct: In the first place, items ceased to operate.
generating revenue are under the control of the ● The Parliament can make laws on any matter in
central government. Thus, the central the State List for implementing the international
government has many revenue sources and the treaties, agreements or conventions. This
States are mostly dependent on the grants and provision enables the Central government to fulfil
financial assistance from the centre. Secondly, its international obligations and commitments.
India adopted planning as the instrument of rapid ● When the President’s rule is imposed in a state,
economic progress and development after the Parliament becomes empowered to make
independence. Planning led to considerable laws with respect to any matter in the State List
centralisation of economic decision making. in relation to that state. A law made so by the
Planning commission appointed by the union Parliament continues to be operative even after
government is the coordinating machinery that the president’s rule.
controls and supervises the resources
management of the States. Besides, the Union Q.59) Ans: b
government uses its discretion to give grants and Exp:
loans to States. ● The Constitution empowers the Centre to
● Statement 2 is correct: The all-India services are exercise control over the state’s legislative
common to the entire territory of India and matters in the following ways:
officers chosen for these services serve in the ● The governor can reserve certain types of bills
administration of the States. Thus, an IAS officer passed by the state legislature for the
who becomes the collector or an IPS officer who consideration of the President. The president
serves as the Commissioner of Police, are under enjoys absolute veto over them.
the control of the central government. States can ● Bills on certain matters enumerated in the State
neither take disciplinary action nor can they List can be introduced in the state legislature only
remove these officers from service. with the previous sanction of the president. (For
example, the bills imposing restrictions on the
Q.58) Ans: d freedom of trade and commerce).
Exp: ● The President can direct the states to reserve
● The Constitution empowers the Parliament to money bills and other financial bills passed by the
make laws on any matter enumerated in the State state legislature for his consideration during a
financial emergency.
● The state legislature has “in normal Secondly, the more important subjects have been
circumstances” exclusive powers to make laws included in the Union List.
with respect to any of the matters enumerated in ● Thirdly, the Centre has overriding authority over
the State List. the Concurrent List. Finally, the residuary powers
● This has at present 61 subjects (originally 662 have also been left with the Centre, while in the
subjects) like public order, police, public health US, they are vested in the states. Thus, the
and sanitation, agriculture, prisons, local Constitution has made the Centre very strong.
government, fisheries, markets, theaters,
gambling and so on. Hence, statement 3 is Q.62) Ans: c
incorrect. Exp:
● Articles 268 to 293 in Part XII of the Constitution
Q.60) Ans: d deal with Centre–state financial relations. Besides
Exp: these, there are other provisions dealing with the
● Federalism is a system of government in which same subject.
the power is divided between a central authority ● Statement 1 is correct: The Constitution also
and various constituent units of the country. draws a distinction between the power to levy
● Statement 1 is incorrect: The Constitution of and collect a tax and the power to appropriate
India, being federal in structure, divides all powers the proceeds of the tax so levied and collected.
(legislative, executive and financial) between the For example, the income-tax is levied and
Centre and the states. However, there is no collected by the Centre but its proceeds are
division of judicial power as the Constitution has distributed between the Centre and the states.
established an integrated judicial system to ● Statement 2 is incorrect: A state legislature can
enforce both the Central laws as well as state impose taxes on the sale or purchase of goods
laws. (other than newspapers). But, this power of the
● Statement 2 is incorrect: The US Model of states to impose sales tax is subjected to the four
federalism is called as coming together as all the restrictions: (a) no tax can be imposed on the sale
states have come together to form a federation. or purchase taking place outside the states; (b) no
However, when a large country, like India, tax can be imposed on the sale or purchase
chooses to divide its authority between taking place in the course of import or Exp:ort; (c)
the member states and the central government, no tax can be imposed on the sale or purchase
it is regarded as holding together federalism. taking place in the course of inter-state trade and
● In this case, power is shared among various social commerce.
groups to accommodate a huge diversity. Here ● Statement 3 is correct: A state legislature can
the central government is more powerful than the impose a tax in respect of any water or electricity
state government. Different constituents of the stored, generated, consumed, distributed or sold
federation may have unequal powers. by any authority established by Parliament for
regulating or developing any inter-state river or
Q.61) Ans: a river valley. But, such a law, to be effective,
Exp: should be reserved for the president’s
● Statement 1 is correct: The process of consideration and receive his assent.
constitutional amendment is less rigid than what
is found in other federations. The bulk of the Q.63) Ans: d
Constitution can be amended by the unilateral Exp:
action of the Parliament, either by simple majority ● Statement 1 is correct: The Constitution
or by special majority. Further, the power to established an integrated judicial system with the
initiate an amendment to the Constitution lies Supreme Court at the top and the state high
only with the Centre. In the US, the states can courts below it. This single system of courts
also propose an amendment to the Constitution. enforces both the Central laws as well as the
● Statement 2 is incorrect: The division of powers is state laws. This is done to eliminate diversities in
in favour of the Centre and highly inequitable the remedial procedure. The judges of a state high
from the federal angle. Firstly, the Union List court are appointed by the president in
contains more subjects than the State List. consultation with the Chief Justice of India and
the governor of the state. They can also be IV. excise duties on alcoholic liquors for human
transferred and removed by the president. consumption and narcotics;
● Statement 2 is correct: During the operation of a V. taxes on the entry of goods into a local area,
national emergency (under Article 352), the on advertisements (except newspapers), on
Centre becomes entitled to give executive consumption or sale of electricity, and on
directions to a state on ‘any’ matter. Thus, the goods and passengers carried by road or on
state governments are brought under the inland waterways;
complete control of the Centre, though they are VI. taxes on professions, trades, callings and
not suspended. employments not exceeding Rs. 2,500 per
annum;
Q.64) Ans: a VII. capitation taxes;
Exp: VIII. tolls;
Centre is empowered to give directions to the States IX. stamp duty on documents (except those
in the execution of following matters: specified in the Union List);
● The construction and maintenance of the means X. sales tax (other than newspaper); and
of communication of national or military XI. fees on the matters enumerated in the
importance by the state. Hence statement 1 is State List (except court fees).
correct.
● The measures to be taken for the protection of Q.66) Ans: c
the railways within the State. Hence statement 3 Exp:
is correct. ● Statement 1 is incorrect: River Board act 1956
● Providing adequate facilities for instruction in the provides for the establishment of river boards to
mother-tongue at the primary stage of education regulate and develop the interstate river and river
to children of linguistic minority groups in the valley
State. o The River Boards Act provides for the
● The drawing up and execution of the schemes establishment of river boards by the Central
essential for the welfare of the Scheduled Tribes government for the regulation and
in the State. Hence statement 2 is not correct. development of inter-state river and river
● The coercive sanction behind the Central valley
directions is Article 365 of Indian Constitution. o The River Board is established on the request
of state governments concerned to advise
Q.65) Ans: b them.
Exp: o The Inter-State Water Disputes Act empowers
● Pair 1 is incorrect: According to Article 268 Taxes the Central government to set up an ad hoc
Levied by the Centre but Collected and tribunal for the adjudication of a dispute
Appropriated by the States includes the following between two or more states in relation to the
taxes and duties: waters of an inter-state river or river valley
o Stamp duties on bills of exchange, cheques, ● Statement 2 is incorrect: AS the river boards are
promissory notes, policies of insurance, transfer established by the central governments on
of shares and others. request of the state governments.
o Excise duties on medicinal and toilet o River Board for advising the Governments
preparations containing alcohol and narcotics. interested in relation to such matters
● Pair 2 is correct: The taxes belonging to the states concerning the regulation or development of an
exclusively are enumerated in the state list and inter- State river or river.
are 20 in number. These are following:
I. land revenue; Q.67) Ans: d
II. taxes on agricultural income, succession and Exp:
estate duties in respect of agricultural land; ● Statement 1 is incorrect : Disputes Resolution
III. taxes on lands and buildings, on mineral Committee: Under the Bill, when a state puts in a
rights, on animals and boats, on road request regarding any water dispute, the central
vehicles, on luxuries, on entertainments, government will set up a Disputes Resolution
and on gambling; Hence, Pair 3 is incorrect. Committee (DRC), to resolve the dispute. DRC
should be set up by the central government not government will set up an Inter-State River
by parliament to resolve disputes amicably. Water Disputes Tribunal, for the adjudication of
o The DRC will comprise a Chairperson, and water disputes. This Tribunal can have multiple
Exp:erts with at least 15 years of Exp:erience in benches. All existing Tribunals will be dissolved,
relevant sectors, to be nominated by the central and the water disputes pending adjudication
government. It will also comprise one member before such existing Tribunals will be transferred
from each state (at Joint Secretary level), who to the new Tribunal.
are party to the dispute, to be nominated by the o Composition of the Tribunal: The Tribunal will
concerned state government. consist of a Chairperson, Vice-Chairperson,
● Statement 2 is incorrect: DRC will seek to resolve three judicial members, and three Exp:ert
the dispute through negotiations ,within one members.
year (extended by six months ,) and submit its o They will be appointed by the central
report to the central government . government on the recommendation of a
o The DRC will seek to resolve the dispute Selection Committee.
through negotiations, within one year o Each Tribunal Bench will consist of a
(extendable by six months), and submit its Chairperson or Vice-Chairperson, a judicial
report to the central government. member, and an Exp:ert member.
o If a dispute cannot be settled by the DRC, the o The central government may also appoint two
central government will refer it to the Inter- Exp:erts serving in the Central Water
State River Water Disputes Tribunal. Such Engineering Service as assessors to advise the
referral must be made within three months Bench in its proceedings.
from the receipt of the report from the DRC. o The assessor should not be from the state which
is a party to the dispute.
Q.68) Ans: c ● Statement 2 is correct : Three judicial members
Exp: and three Exp:ert members are part of the
● Statement 1 is incorrect : Dispute Resolution tribunal.
Committee (DRC) will comprise a chairperson and ● Statement 3 is Incorrect : Tribunal members are
does not consist of vice chairperson .The DRC will appointed by the central government on the
comprise a Chairperson, and Exp:erts with at recommendation of the selection committee.
least 15 years of Exp:erience in relevant sectors,
to be nominated by the central government. Q.70) Ans: b
● Statement 2 is incorrect : Dispute Resolution Exp:
Committee (DRC) DRC does not comprise any ● Statement 1 is incorrect: The Inter-State Council
judicial member. is a non-permanent constitutional body enshrined
● Statement 3 is correct : Exp:erts with at least 15 in Article 263 of the Constitution of India. It can
years of Exp:erience in relevant sectors,to be set up the President based on the provisions of
nominated by a Central Government are present Article 263. First, such a body was established in
in DRC. 1990 on the recommendation of the Government
• And one member from each state who will be Commission under the Ministry of Home Affairs.
nominated by the concerned State Government Inter state council (ISC ) does not interrupt in
,party to the dispute. court proceedings regarding all india services.
o It is important to note that the very first reason
Q.69) Ans: a the Constitution gives for setting up the
Exp: institution is that it will be useful when it comes
● Statement 1 is correct: The Inter-State River to inquiring into and advising upon disputes
Water Disputes (Amendment) Bill, 2019 was which may have arisen between states.
introduced in Lok Sabha on July 25, 2019 by the o The other two are to discuss subjects for which
Minister of Jal Shakti, Mr. Gajendra Singh the components of the Indian union have
Shekhawat. It amends the Inter-State River Water common interests and to figure out how to
Disputes Act, 1956. The Act provides for the coordinate policy
adjudication of disputes relating to waters of ● Statement 2 is correct: Duties to investigate and
inter-state rivers and river valleys. The central discuss the subjects of common interest between
union and states and better coordination of policy
. • Pair 1 is incorrect: Following days of violent
o Deliberate on such matters of general interest protests across Kazakhstan, a significant and
to the States referred by the Chairman to the
Council. It shall have its own Secretariat.
o Inquiring into and advising upon disputes which
may have arisen between states;
o Investigating and discussing subjects in which
some or all of the States or the Union and one
or more of the states, have a common interest.

Q.71) Ans: b
Exp:
• The ambitious Creating Opportunities for
Manufacturing, Preeminence in Technology, and
Economic Strength (COMPETES) Act of 2022,
which seeks to open up new vistas for brilliant sudden increase in fuel prices caused a national
individuals from around the world with a new crisis, with the government officially stepping
start-up visa, has been launched by the United down. Angry Kazakhs initially flocked to the
States. It aspires to strengthen supply chains and streets after the government loosened price
re-energize the country's innovation engine so limitations on LPG, causing gasoline prices to
that it can compete with China and the rest of the quadruple in the oil-rich Central Asian nation. The
globe for decades to come. demonstrations began in the oil city of
• The Act exempts STEM (science, technology, Zhanaozen, where police killed at least 16 oil
engineering, or mathematics) PhDs from the employees protesting bad working conditions in
green card cap and creates a special green card 2011.
specifically for entrepreneurs. A Green Card
holder (permanent resident) is a person who has • Pair 2 is correct: According to satellite pictures,
been granted permission to live and work North Korea appears to have begun renovating a
permanently in the United States. The act decommissioned nuclear test site. After leader
allocates USD 600 million per year to create Kim Jong-un committed to stop all nuclear tests,
manufacturing facilities in the United States, the Punggye-ri site was shut down in 2018 and
reducing the country's reliance on solar some components were blown up. The new
components made in Xinjiang, China. conclusion by US analysts comes amid speculation
• It introduces a new "W" nonimmigrant that the country may resume nuclear and long-
classification for entrepreneurs with a stake in a range missile testing. However, analysts estimate
startup, vital employees of a startup, and their that Punggye-ri will take years to complete.
spouses and children.

Q.72) Ans: b
Exp:
• Pair 3 is correct: Thousands of people came to the and Central Asia would be established in New
streets of Sudan's capital and other cities in a Delhi to support the new structure.
continuation of the country's unending protests
that have rocked the country since a military coup Q.74) Ans: d
in October 2021. Since the military took over on Exp:
October 25, 2021, at least 76 people have died as
a result of the protests. The new crackdown is
Exp:ected to make it more difficult for the United
Nations to find a way out of the country's
predicament. According to the pro-democracy
movement, protesters, largely young people,
marched through the streets of Khartoum and its
sister city, Omdurman. Protests were also held in • Pair 1 is correct: The India-assisted Social
other parts of the country, including Kassala, Red Housing Units project in Mauritius was virtually
Sea, Jazira, and the already tense Darfur region, inaugurated on January 20 by Prime Minister
according to the organization. Narendra Modi and his Mauritius counterpart
Pravind Kumar Jugnauth. The Social Housing
Q.73) Ans: d Project at Dagotiere and Mare Tabac was
Exp: established with the intention of providing low-

cost housing for Mauritius' disadvantaged


households. The Government of India supported
the 956 housing units project, which was finished
on time and on budget at a cost of 45 million
dollars. Roads, water supply, sewerage, drains,
• The first India-Central Asia Summit in virtual and other public services are all part of the
format was recently convened by India's Prime project's infrastructure.
Minister. Presidents of Kazakhstan, Kyrgyzstan,
Tajikistan, Turkmenistan, and Uzbekistan were
among those who attended.
• This was the first India-Central Asia summit,
which took place on the 30th anniversary of the
two nations' diplomatic relations. They talked
about how to take India-Central Asia relations to
new heights in the future. The Leaders decided in
a historic move to institutionalize the Summit
process by deciding to conduct it every two years.
They also decided to hold regular meetings of
Foreign Ministers, Trade Ministers, Culture
Ministers, and Security Council Secretaries to lay
the basis for the Summits. A secretariat for India
• Pair 2 is correct: An alleged drone strike in Abu China, Indonesia, Lao PDR, Malaysia, Myanmar,
Dhabi, the capital of the United Arab Emirates Nepal, Russia, Thailand and Vietnam.
(UAE), recently resulted in numerous Exp:losions, • CA|TS is a globally accepted conservation tool
killing two Indians. The strike was claimed by that sets best practice and standards to manage
Yemen's Shia Houthi rebels, who have been in tigers and encourages assessments to benchmark
charge of the country's northern regions, progress. Habitats which support tiger
including the capital Sana'a, for seven years. In populations are the building blocks of wild tiger
the face of such heinous atrocities, India has conservation and effectively managing them is
Exp:ressed its sympathy with the UAE. The essential for long-term survival of wild tigers.
Houthis launched a ballistic missile attack on • The 14 tiger reserves, which have been accredited
Riyadh, Saudi Arabia's capital, earlier in 2021. are:
o Manas, Kaziranga and Orang in Assam
o Satpura, Kanha and Panna in Madhya Pradesh
o Pench in Maharashtra
o Valmiki Tiger Reserve in Bihar
o Dudhwa in Uttar Pradesh
o Sunderbans in West Bengal
o Parambikulam in Kerala
o Bandipur Tiger Reserve in Karnataka
o Mudumalai and Anamalai Tiger Reserves in
Tamil Nadu.

Q.76) Ans: b
Exp:
• Option A is incorrect: The Beijing Declaration and
Platform for Action is an agenda for women’s
• Pair 3 is correct: Burkina Faso's army recently
empowerment. It seeks to promote equality
stated that President Roch Kabore had been
between women and men as a matter of human
deposed, that the constitution had been
rights and a condition for social justice and is also
suspended, that the government and national
a necessary and fundamental prerequisite for
legislature had been dissolved, and that the
equality, development and peace.
country's borders had been blocked. In Mali and
• Option B is correct: During the St. Petersburg
Guinea, the army has overthrown administrations
Declaration in 2010, the countries agreed to a
in the last 18 months. After President Idriss Deby
Global Tiger Recovery Program and pledged to
died fighting insurgents on the battlefield in
double the global tiger population by 2022.
Chad's north last year (2021), the military took
• Option C is incorrect: On 30 January 2012,
command.
pharmaceutical companies, donors, endemic
countries and non-government organizations
Q.75) Ans: c
came together to sign the London Declaration on
Exp:
Neglected Tropical Diseases. Together, they
• Option C is correct: Conservation Assured | Tiger
committed to control, eliminate or eradicate 10
Standards (CA|TS) has been agreed upon as an
diseases by 2020 and improve the lives of over a
accreditation tool by the global coalition of Tiger
billion people.
Range Countries (TRCs) and has been developed
• Option D is incorrect: The Brazzaville Declaration
by tiger and protected area Exp:erts. Officially
aims to implement coordination and cooperation
launched in 2013, it sets minimum standards for
between different government sectors to protect
effective management of target species and
the benefits provided by peatland ecosystems.
encourages assessment of these standards in
The Brazzaville Declaration promotes better
relevant conservation areas. CA|TS is a set of
management and conservation of the peatlands,
criteria which allows tiger sites to check if their
which represent one of the biggest carbon stores
management will lead to successful tiger
on the planet, for climate mitigation and other
conservation. There are currently 13 tiger range
benefits.
countries - India, Bangladesh, Bhutan, Cambodia,
Q.77) Ans: b (79.33%), Meghalaya (76.00%), Manipur (74.34%)
Exp: and Nagaland (73.90%).
• Statement 1 is incorrect: 'India State of Forest
Report 2021’ is prepared by the Forest Survey of Q.78) Ans: c
India (FSI) which has been mandated to assess the Exp:
forest and tree resources of the country. In 2021, • Statement 1 is correct:Recently, the National
the total forest and tree cover in India is 80.9 Green Tribunal (NGT) has directed Delhi and
million hectares, which is 24.62% of the Haryana to enforce the Environment
geographical area of the country. As compared to Management Plan (EMP) that the two
the assessment of 2019, there is an increase of governments have prepared for the rejuvenation
2,261 sq km in the total forest and tree cover of and protection of the Najafgarh Jheel, a
the country. Out of this, the increase in the forest transboundary wetland. Najafgarh Jheel is located
cover has been observed as 1,540 sq km and that in a natural depression in southwest Delhi, close
in tree cover is 721 sq km. to the Gurugram-Rajokri border on National
o 17 states/UTs have above 33% of the Highway-48. Najafgarh Jheel is a transboundary
geographical area under forest cover. Out of wetland. It is located along the borders of Delhi
these states and UT’s, five states/UTs namely and Haryana. The Najafgarh lake is under the
Lakshadweep, Mizoram, Andaman & Nicobar control of Delhi.
Islands, Arunachal Pradesh and Meghalaya have • Statement 2 is correct: The lake is largely filled
more than 75 percent forest cover while 12 with sewage from Gurugram and surrounding
states/UTs namely Manipur, Nagaland, Tripura, villages of Delhi. A portion of the lake falls in
Goa, Kerala, Sikkim, Uttarakhand, Chhattisgarh, Haryana. The presence of 281 bird species,
Dadra & Nagar Haveli and Daman & Diu, Assam, including several threatened ones such as
Odisha, have forest cover between 33 percent Egyptian vulture, Sarus Crane, Steppe Eagle,
to 75 percent. Greater Spotted Eagle, Imperial Eagle and those
• Statement 2 is correct: Increase in forest cover migrating along the Central Asian Flyway has been
has been observed in open forest followed by reported at the lake
very dense forest. Top three states showing • Statement 3 is incorrect: The Sahibi river, also
increase in forest cover are Andhra Pradesh (647 called the Sabi River, is an ephemeral, rain-fed
sq km) followed by Telangana (632 sq km) and river flowing through Rajasthan, Haryana and
Odisha (537 sq km). Delhi states in India. It drains into Yamuna in
o Total mangrove cover in the country is 4,992 sq Delhi, where its channeled course is also called
km. An increase of 17 sq Km in mangrove cover the Najafgarh drain, which also serves as
has been observed as compared to the previous Najafgarh drain bird sanctuary.
assessment of 2019. Top three states showing
mangrove cover increase are Odisha (8 sq km) Q.79) Ans: d
followed by Maharashtra (4 sq km) and Exp:
Karnataka (3 sq km). • Statement 1 is correct: Rogue, freak, or killer
o Total carbon stock in country’s forest is waves are often called 'extreme storm waves' by
estimated to be 7,204 million tonnes and there scientists, are those waves which are greater than
an increase of 79.4 million tonnes in the carbon twice the size of surrounding waves. They are very
stock of country as compared to the last unpredictable, and often come unExp:ectedly
assessment of 2019. The annual increase in the from directions other than prevailing wind and
carbon stock is 39.7 million tonnes. waves. They look like "walls of water." They are
o Area-wise Madhya Pradesh has the largest often steep-sided with unusually deep troughs.
forest cover in the country followed by • Statement 2 is correct: Causes of their
Arunachal Pradesh, Chhattisgarh, Odisha and occurrence - Focusing of wave energy: When
Maharashtra. waves formed by a storm develop in a water
• Statement 3 is correct: In terms of forest cover as current against the normal wave direction, an
percentage of total geographical area, the top five interaction can take place which results in a
States are Mizoram (84.53%), Arunachal Pradesh shortening of the wave frequency. This can cause
the waves to dynamically join together, forming
very big 'rogue' waves. The currents where these provide speedy environmental justice and help
are sometimes seen are the Gulf Stream and reduce the burden of litigation in the higher
Agulhas current. courts. The Tribunal is mandated to make and
• Statement 3 is correct: Constructive interference: endeavor for disposal of applications or appeals
Extreme waves often form because swells, while finally within 6 months of filing of the same. New
traveling across the ocean, do so at different Delhi is the Principal Place of Sitting of the
speeds and directions. As these swells pass Tribunal and Bhopal, Pune, Kolkata and Chennai
through one another, their crests, troughs, and are the other four places of sitting of the Tribunal.
lengths sometimes coincide and reinforce each
other. This process can form unusually large, Q.81) Ans: d
towering waves that quickly disappear. If the Exp:
swells are traveling in the same direction, these • Option D is correct: Recently, the National Green
mountainous waves may last for several minutes Tribunal (NGT) directed the constitution of a ‘Fly
before subsiding. Ash Management and Utilisation Mission’. The
Fly Ash Management and Utilisation Mission,
Q.80) Ans: c besides monitoring the disposal of annual stock of
Exp: unutilised fly ash, will also see how 1,670 million
• Statement 1 is correct: The National Green tonnes of legacy (accumulated) fly ash could be
Tribunal has been established under the National utilized in the least hazardous manner and how all
Green Tribunal Act, 2010 for effective and safety measures could be taken by the power
Exp:editious disposal of cases relating to plants.
environmental protection and conservation of o Fly ash is an unwanted unburnt residue of coal
forests and other natural resources including combustion in a coal thermal power plant. It is
enforcement of any legal right relating to emitted along with flue gases during the
environment and giving relief and compensation burning of coal in a furnace and collected using
for damages to persons and property and for the electrostatic precipitators. The fly ash
matters connected therewith or incidental collected with the help of precipitators is
thereto. The Tribunal shall not be bound by the converted into a wet slurry to minimise
procedure laid down under the Code of Civil fugitive dust emissions. It is then transported to
Procedure, 1908, but shall be guided by principles the scientifically designed ash ponds through
of natural justice. With the establishment of the slurry pipelines. Fly ash includes substantial
NGT, India became the third country in the world amounts of silicon dioxide (SiO2), aluminium
to set up a specialized environmental tribunal, oxide (Al2O3), ferric oxide (Fe2O3) and calcium
only after Australia and New Zealand, and the first oxide (CaO).
developing country to do so.
• Statement 2 is incorrect: Members are chosen by Q.82) Ans: b
a selection committee (headed by a sitting judge Exp:
of the Supreme Court of India) that reviews their • Pair 1 is incorrect: The spot-billed pelican
applications and conducts interviews. (Pelecanus philippensis) or grey pelican is a
The Judicial members are chosen from applicants member of the pelican family. It breeds in
who are serving or retired judges of High Courts. southern Asia from southern Iran across India east
Exp:ert members are chosen from applicants who to Indonesia. It is a bird of large inland and coastal
are either serving or retired bureaucrats not waters, especially large lakes. The breeding
below the rank of an Additional Secretary to the population of these pelican species is limited to
Government of India (not below the rank of India, Sri Lanka and Cambodia. In the non-
Principal Secretary if serving under a state breeding season they are recorded in Nepal,
government) with a minimum administrative Myanmar, Thailand, Laos and Vietnam.
Exp:erience of five years in dealing with o Conservation status:
environmental matters. Or, the Exp:ert members IUCN status: Near Threatened
must have a doctorate in a related field. Wildlife (Protection) Act, 1972: Schedule IV
• Statement 3 is correct: The Tribunal's dedicated (Hunting prohibited but the penalty for any
jurisdiction in environmental matters shall
violation is less compared to the first two Q.84) Ans: a
schedules). Exp:
• Pair 2 is correct: India has achieved a remarkable • Option A is correct: The creation of an Indian
feat by doubling the tiger population four years Environmental Service (IES) was recommended
ahead of its goal. Conservation Assured |Tiger by a committee headed by former Cabinet
Standards (CA|TS) accreditation has already been secretary T.S.R Subramanian in 2014. The
awarded to 14 tiger reserves in India. India has committee was established to review
successfully translocated a tiger in the western environmental laws in the country and to bring
part of the Rajaji Tiger Reserve. Similar them in line with the then required needs.
reintroduction has been planned for Kanha Tiger o Observations of TSR Subramanian Committee:
Reserve in Madhya Pradesh. Although India had a strong environmental
o Conservation Status of Tiger: policy and legislative framework, there was no
Indian Wildlife (Protection) Act, 1972: Schedule effective coordination amongst various
I Ministries/institutions regarding the integration
IUCN status: Endangered. of environmental concerns. There is a lack of
Convention on International Trade in trained personnel involved in the
Endangered Species of Wild Fauna and Flora administration, policy formulation, and
(CITES): Appendix I. supervising the implementation of policies of
• Pair 3 is incorrect: There are three subspecies of the state and central governments. Looking at
swamp deer found in the Indian Subcontinent. the current administrative set-up, it can be
The western swamp deer (Rucervus duvaucelii) inferred that the government servants might
found in Nepal. Southern swamp deer/Hard not be able to spare special time for
Ground Barasingha (Rucervus duvaucelii branderi) environmental causes.
found in central and north India. Eastern swamp • Option B is incorrect: Swaminathan committee
deer (Rucervus duvaucelii ranjitsinhi) found in the was constituted to find the problems faced by the
Kaziranga (Assam) and Dudhwa National Parks farmers. The Swaminathan Report covers key
(Uttar Pradesh). recommendations and findings related to farmer
o Protection Status of Swamp Deer: competitiveness, productivity of agriculture,
IUCN Red List: Vulnerable irrigation, employment, food security, credit and
CITES: Appendix I insurance.
Wildlife Protection Act, 1972: Schedule I. • Option C is incorrect: Lodha committee was
constituted to recommend reforms for cricket in
Q.83) Ans: d India.
Exp: • Option D is incorrect: Arun Goel committee was
• Option D is correct: Nandankanan Zoological constituted to strengthen the Capital Goods(CG)
Park is located in Odisha. It was inaugurated in Sector and contribute more actively in the
1960. It is the first zoo in the country to become a national goal of achieving a USD 5 trillion
member of the World Association of Zoos & economy and a USD 1 trillion manufacturing
Aquariums (WAZA). WAZA is the global alliance of sector.
regional associations, national federations, zoos
and aquariums, dedicated to the care and Q.85) Ans: b
conservation of animals and their habitats around Exp:
the world. It is recognized as a leading zoo for the • Statement 1 is incorrect: State Environment
breeding of the Indian pangolin and white tiger. Impact Assessment Authority (SEIAAs) is a very
Leopards, mouse deer, lions, rats and vultures are important arm of the Ministry for implementation
also bred here. It was the world’s first captive of Environmental Impact Assessment (EIA)
crocodile breeding center, where gharials were Notification at the State level and they have been
bred in captivity in 1980. The State Botanical delegated powers to consider and grant
Garden of Nandankanan is one of the pioneering environmental clearance (EC) for all proposals
plant conservation and nature education centers under Category B.
of Odisha. • Statement 2 is correct: There are seven criteria to
rate State Environmental Impact Assessment
Agencies (SEIAA) on “transparency, efficiency and known for its therapeutic properties and is in high
accountability”. On a scale of seven, a SEIAA, for demand for its cosmetic and medicinal properties.
instance, gets two marks for granting a clearance It is also used to make furniture and demand a
in less than 80 days, one mark for within 105 days high value in the international market. Its
and no marks for more. If less than 10% of the popularity can be judged from the fact that a
projects for scrutiny prompted a site visit by tonne of Red Sanders costs anything between Rs
committee members, to examine ground 50 lakh to Rs 1 crore in the international market.
conditions, a SEIAA would get one mark. More o Threats: Illicit felling for smuggling, forest fires,
than 20%, on the other hand, would be a demerit cattle grazing and other anthropogenic threats.
or zero marks. SEIAA with a score of seven or • Statement 3 is correct: Protection Status- IUCN
more would be rated ‘five star.’ Red List: Endangered, CITES: Appendix II, Wildlife
• Statement 3 is incorrect: There is no negative (Protection) Act 1972: Schedule II
marking proposed for not meeting the criteria for
ranking. Q.88) Ans: a
Exp:
Q.86) Ans: c • Option A is correct: Miss Kerala, also known as
Exp: Denison barb, red-line torpedo barb and roseline
• Statement 1 is correct: Pangolins are scaly shark. It is a fish species with red and black stripes
anteater mammals of the order Pholidota. They on its body. Its scientific name is Sahyadria
have large, protective keratin scales covering their denisonii. It is found in the States of Kerala and
skin. They are the only known mammals on earth Karnataka. Its IUCN Status is Endangered. This
to have this feature. Pangolins are nocturnal, and species is known to inhabit fast-flowing hill
their diet consists of mainly ants and termites, streams and is often found in rocky pools with
which they capture using their long tongues.I thick vegetation along river banks. The fish
• Statement 2 is incorrect: Out of the eight species species is being Exp:loited for the aquarium trade.
of pangolin, the Indian Pangolin and the Chinese • Option B is incorrect: It has been named after
Pangolin are found in India. The distribution of plant taxonomist C.N. Sunil, retired professor and
Indian Pangolin is wide in India, except in the arid research guide of Botany, SNM College. It is a
region, high Himalayas and the North-East. The perennial plant of the Cyperaceae family. It stands
species also occurs in Bangladesh, Pakistan, Nepal 20-59 cm tall and was collected from an elevation
and Sri Lanka. of 1,100 meters in Western Ghats region. It has
• Statement 3 is correct: Indian Pangolin - IUCN been provisionally assessed as data deficient (DD)
Status: Endangered. Wildlife Protection Act,1972: under the IUCN Red List categories.
under Schedule I. • Option C is incorrect: Researchers from Kerala
o Chinese Pangolin - Distribution of Pangolins have identified two new species of fungi from the
happens widely in Vietnam, Thailand, Cambodia genus Ganoderma that are associated with
and the northeastern part of India. IUCN Status: coconut stem rot disease. They have also
Critically Endangered, Wildlife Protection genotyped the two fungi species, named
Act,1972: Under Schedule I. Ganoderma keralense and G. pseudoapplanatum
and identified genetic biomarkers.
Q.87) Ans: d o Butt rot or Basal stem rot of Coconut: The basal
Exp: stem rot of coconut is known by several names
• Statement 1 is correct: Red Sanders(Pterocarpus in different parts of India: Ganoderma wilt in
santalinus) is an Indian endemic tree species with Andhra Pradesh, Anaberoga in Karnataka,
a restricted geographical range in the Eastern Thanjavur wilt in Tamil Nadu. The infection
Ghats. The species is endemic to a distinct tract of begins at the roots. The symptoms include
forests in Andhra Pradesh. Some contiguous discolouration and rotting of stems and leaves.
patches in Tamil Nadu and Karnataka also see A reddish-brown secretion is seen and this
some wild growth. secretion is reported only in India. Once
• Statement 2 is correct: Red Sanders usually grow infected, recovery of the plants is highly
in the rocky, degraded and fallow lands with Red unlikely. In the final stages, the flowering
Soil and hot and dry climate. Red Sanders is decreases and the coconut palm dies.
• Option D is incorrect: It is a prostrate perennial • Option A is incorrect: In an autologous
herb named after K.M. Prabhukumar in transplant, your own blood-forming stem cells are
recognition of his research on flowering plants of collected. You are then treated with high doses of
the Western Ghats. It was discovered in the chemotherapy. The high-dose treatment kills the
Chembra Peak grasslands of Wayanad. It hails cancer cells, but it also gets rid of the blood-
from the family Rubiaceae and grows on high- producing cells that are left in your bone marrow.
altitude grasslands. It grows up to 70 cm in length Afterward, the collected stem cells are put back
and is many-flowered with the petals pale pink in into your bloodstream, allowing the bone marrow
colour. to produce new blood cells.
• Option B is incorrect: An allogeneic stem cell
Q.89) Ans: b transplant uses healthy blood stem cells from a
Exp: donor to replace your diseased or damaged bone
• Statement 1 is incorrect: Millimetre Wave-Band marrow. An allogeneic stem cell transplant is also
is a particular segment of the radio frequency called an allogeneic bone marrow transplant.
spectrum that ranges between 24 GHz and 100 • Option C is correct: Xenotransplantation is “any
GHz. This spectrum has a short wavelength, and is procedure that involves the transplantation,
appropriate to deliver greater speeds and lower implantation or infusion into a human recipient of
latencies. This in turn makes data transfer either (a) live cells, tissues, or organs from a
efficient and seamless as the current available nonhuman animal source, or (b) human body
networks work optimally only on lower frequency fluids, cells, tissues or organs that have had ex
bandwidths. vivo contact with live nonhuman animal cells,
o The current available networks work optimally tissues or organs”. Xenotransplantation is seen as
well on the shorter wavelengths, but they don’t an alternative to the clinical transplantation of
work well on the parameter of speed, which is human organs whose demand around the world
needed for 5G. exceeds supply by a long distance.
o So for 5G to work at its full potential, and speed • Option D is incorrect: A procedure in which a
up the mm wave band is very essential. patient receives healthy stem cells (blood-forming
o The mm-band had been the subject of cells) to replace their own stem cells that have
controversy due to out-of-band emissions into been destroyed by treatment with radiation or
the passive satellite band used for weather high doses of chemotherapy. In a syngeneic bone
satellites at 23.6-24 GHz. Out-of-band emission marrow transplant, the healthy stem cells come
is emission on a frequency or frequencies from the bone marrow of the patient’s healthy
immediately outside the necessary bandwidth identical twin.
which results from the modulation process.
• Statement 2 is correct: Millimeter waves are Q.91) Ans: a
absorbed by gases and moisture in the Exp:
atmosphere, which reduces the range and • Statement 1 is correct: A solid-state battery has
strength of the waves. Rain and humidity reduce higher energy density than a Lithium-ion battery
their signal strength and propagation distance, a that uses liquid electrolyte solution. It doesn’t
condition known as rain fade. The propagation have a risk of Exp:losion or fire, so there is no
distance at the lower frequencies is up to a need to have components for safety, thus saving
kilometer, while the higher frequencies travel more space. Then we have more space to put
only a few meters. A millimeter wave travels by more active materials which increase battery
line of sight and is blocked or degraded by capacity in the battery. A solid-state battery can
physical objects like trees, walls and buildings. Its increase energy density per unit area since only a
propagation is also affected by proximity to small number of batteries are needed. The
humans and animals, primarily due to their water advantages of the solid-state battery technology
content. include higher cell energy density (by eliminating
the carbon anode), lower charge time (by
Q.90) Ans: c eliminating the need to have lithium diffuse into
Exp: the carbon particles in conventional lithium-ion
cells), ability to undertake more charging cycles space than the speed of light. This means a
and thereby a longer life, and improved safety. black hole's event horizon is essentially the
• Statement 2 is incorrect: Solid-state batteries can point from which nothing can return. The name
provide solutions for many problems of liquid Li- refers to the impossibility of witnessing any
ion batteries, such as flammability, limited event taking place inside that border, the
voltage, unstable solid-electrolyte interphase horizon beyond which one cannot see.
formation, poor cycling performance and o Within the event horizon, one would find the
strength. In the charging & discharging cycle, ions black hole's singularity, where previous research
transfer to and fro between the anode (negative suggests all of the object's mass has collapsed
electrode generally made of graphite) and to an infinitely dense extent. This means the
cathode (positive electrode made of lithium). fabric of space and time around the singularity
has also curved to an infinite degree, so the
Q.92) Ans: a laws of physics as we currently know them
Exp: break down.
• Statement 1 is correct: Dark matter is composed
of particles that do not absorb, reflect, or emit Q.94) Ans: b
light, so they cannot be detected by observing Exp:
electromagnetic radiation. A dark matter halo is a • Option B is correct: X-ray Polarimeter Satellite
basic unit of cosmological structure. It is a (XPoSat) is an ISRO planned space observatory to
hypothetical region that has decoupled from study the polarization of cosmic X-rays. It aims to
cosmic Exp:ansion and contains gravitationally study various dynamics of astronomical sources in
bound matter. Thought to consist of dark matter, extreme conditions. XPoSat will study the 50
halos have not been observed directly. Their brightest known sources in the universe, including
existence is inferred through observations of their pulsars, black hole X-ray binaries, active galactic
effects on the motions of stars and gas in galaxies nuclei, and nonthermal supernova remnants.
and gravitational lensing. • The spacecraft will carry two scientific payloads in
• Statement 2 is correct: Dark matter is material a low earth orbit with preference for a low
that cannot be seen directly and it has never been inclination orbit. The primary payload POLIX
detected directly. We know that dark matter (Polarimeter Instrument in X-rays) will measure
exists because of the effect it has on objects that the polarimetry parameters (degree and angle of
we can observe directly. polarization) of astronomical sources in medium
• Statement 3 is incorrect: Roughly 68% of the X-ray energy of 8-30 keV photons. The XSPECT (X-
universe is dark energy. Dark matter makes up ray Spectroscopy and Timing) payload will give
about 27%. The rest - everything on Earth, spectroscopic information of soft X-rays in the
everything ever observed with all of our energy range of 0.8-15 keV.
instruments, all normal matter - adds up to less • The spacecraft is planned to be launched in 2021
than 5% of the universe. but due to the Chinese Wuhan virus pandemic the
launch may get delayed. XPOSAT will provide a
Q.93) Ans: b service time of at least five years. The observatory
Exp: will be placed in a circular low Earth orbit of 500
• Option B is correct: The event horizon of a black to 700 km.
hole is linked to the object's escape velocity - the
speed that one would need to exceed to escape Q.95) Ans: d
the black hole's gravitational pull. The closer Exp:
someone came to a black hole, the greater the • Option A is correct: Open Source Software refers
speed they would need to escape that massive to publicly accessible software.
gravity. The event horizon is the threshold around • Option B is correct: Source code is the basic code
the black hole where the escape velocity on which the software is formulated. In open
surpasses the speed of light. The size of an event source softwares, source code is available for
horizon depends on the black hole's mass. public to manipulate and modify.
o According to Einstein's theory of special • Option C is correct: It includes a license that
relativity, nothing can travel faster through allows programmers to modify the software as
per their needs. It works on the following rules. There is no need for a central authority to
underlying Principles: Open exchange, authenticate or validate.
Collaborative participation, Rapid • Statement 2 is correct: With block chain, the
prototyping, Transparency, Meritocracy and time and place of the transaction are recorded
Community-oriented development. permanently. Thus, Web3 enables peer to peer
• Option D is incorrect: Unlike source code of (seller to buyer) transactions by eliminating the
closed softwares which are encrypted, OSS code is role of the intermediary.
publicly available. • Statement 3 is correct: In Web3, users will have
ownership stakes in platforms and applications
Q.96) Ans: c unlike now where tech giants control the
Exp: platforms. Web3 will deliver a decentralized and
• Statement 1 is correct: Negative ion technology fair internet where users control their own data.
embeds negative ions in personal products and is
currently being advertised as a means to maintain Q.98) Ans: a
health, balance energy, and improve well-being. Exp:
This technology is used in certain silicone • Statement 1 is correct: The AT4 is a single-shot,
wristbands, quantum or scalar-energy pendants, recoilless smoothbore anti-tank weapon that is
and kinesthesiology tape. Negative ions are also unguided, man-portable, and disposable. It is one
made when sunlight, radiation, air, or water break of the world's most widely used light antitank
down oxygen. The minerals that produce these weapons. They're small, single-shot, and
negative ions often include naturally occurring completely disposable, and they're known for
radioactive substances such as uranium and their ease of use and handling. The weapon is
thorium. It is believed that negative ions create around 9 kg in weight and has a range of 200
positive vibes and uplift the mood. They show the metres. The AT4 is intended to give infantry units
various mental and physical health benefits, such a means to destroy or disable armoured vehicles
as stress reduction, better sleeping, respiration and fortifications
etc. whereas these ions may also act on • Statement 2 is incorrect: Major Saab, a Swedish
pollutants, make them negatively charged and get defence company, manufactures the AT4. The
them collected on surfaces. AT4 is designed to allow infantry units to attack or
• Statement 2 is correct: Exp:osure to ionizing disable fortifications and armoured vehicles. The
radiation can cause adverse health effects and launcher and projectile are packaged together
wearing the products for extended periods could and issued as a single unit of ammunition, with
pose health risks that include tissue and DNA the launcher being discarded after each use.
damage. Exp:osure can also cause severe harmful
effects such as: Skin burns, Acute radiation Q.99) Ans: a
sickness that causes cancer and hairfall, Exp:
Temporary reduction in white blood cells, Possible • Statement 1 is correct: The Multi-Agency Centre
chromosomal damage, Reduction in resistance to (MAC) is a central location where numerous
infection. agencies can share intelligence inputs. The MAC is
made up of around 28 agencies, and it includes
Q.97) Ans: b every organisation that is involved in counter-
Exp: terrorism in any form. The Research and Analysis
• Statement 1 is incorrect: Web 3.0 is a Wing (R&AW), the armed forces, and state police
decentralized internet to be run on blockchain are all examples of this. On the MAC, many
technology, which would be different from the security agencies share real-time intelligence
versions in use, Web 1.0 and Web 2.0. The spirit information.
of Web3 is Decentralized Autonomous • Statement 2 is incorrect: The Intelligence Bureau
Organization (DAO). DAO is all about the business (IB) is the MAC's nodal agency. At the MAC level,
rules and governing rules in any transaction are a conference of all of these agencies is held
transparently available for anyone to see and practically every day, during which the
software will be written conforming to these intelligence from the previous 24 hours is brought
to the table, discussed, and a plan of action
established or agreed upon.
• Statement 3 is incorrect: After the Kargil
invasion, the national-level MAC was established
in Delhi in 2001. Subsidiary MACs exist in the
states as well (SMACs). The MAC headquarters is
connected to around 400 secure sites. There has
also been talk about Exp:anding SMAC
connectivity to the district level.

Q.100) Ans: b
Exp:
• Statement 1 is incorrect: After the assassination
of former Prime Minister Indira Gandhi in 1985,
the Special Protection Group (SPG) was formed. It
was established with the purpose of providing
personal security to the Prime Minister, former
Prime Ministers, and their closest family
members.
• Statement 2 is correct: The law was revised in
2019, and SPG protection is now solely
guaranteed to the Prime Minister of India. Men
and women from the Central Armed Police Forces
(such as the Border Security Force, Central
Reserve Police Force, Central Industrial Security
Force, and the IndoTibetan Border Police) and the
Railway Protection Force Service make up the
SPG. SPG is in charge of the Prime Minister's
safety and security. The Prime Minister is always
protected by the SPG, both in India and overseas.

You might also like